Question and Answers Forum

All Questions      Topic List

Arithmetic Questions

Previous in All Question      Next in All Question      

Previous in Arithmetic      Next in Arithmetic      

Question Number 99603 by bemath last updated on 22/Jun/20

How many terms in the geometric  progression 1, 1.1, 1.21,1.331,...  will be needed so that the sum of  the first n terms is greather than 20?

$$\mathrm{How}\:\mathrm{many}\:\mathrm{terms}\:\mathrm{in}\:\mathrm{the}\:\mathrm{geometric} \\ $$$$\mathrm{progression}\:\mathrm{1},\:\mathrm{1}.\mathrm{1},\:\mathrm{1}.\mathrm{21},\mathrm{1}.\mathrm{331},... \\ $$$$\mathrm{will}\:\mathrm{be}\:\mathrm{needed}\:\mathrm{so}\:\mathrm{that}\:\mathrm{the}\:\mathrm{sum}\:\mathrm{of} \\ $$$$\mathrm{the}\:\mathrm{first}\:{n}\:\mathrm{terms}\:\mathrm{is}\:\mathrm{greather}\:\mathrm{than}\:\mathrm{20}? \\ $$

Commented by john santu last updated on 22/Jun/20

The sequence is a geometric progression  with  { ((a=1)),((r=1.1)) :}  we want to find the smallest value of  n such that S_n  > 20  ⇒S_n  = ((a(1−r^n ))/(1−r)) = ((1.(1−(1.1)^n )/(1−1.1)) > 20  ((1−(1.1)^n )/(−0.1)) > 20 ; (1.1)^n −1 > 2  (1.1)^n  > 3 ; n > ((ln 3)/(ln 1.1)) ; n > 11.526  and therefore the smallest number   value of n is 12 ■

$$\mathrm{The}\:\mathrm{sequence}\:\mathrm{is}\:\mathrm{a}\:\mathrm{geometric}\:\mathrm{progression} \\ $$$$\mathrm{with}\:\begin{cases}{{a}=\mathrm{1}}\\{{r}=\mathrm{1}.\mathrm{1}}\end{cases} \\ $$$$\mathrm{we}\:\mathrm{want}\:\mathrm{to}\:\mathrm{find}\:\mathrm{the}\:\mathrm{smallest}\:\mathrm{value}\:\mathrm{of} \\ $$$${n}\:\mathrm{such}\:\mathrm{that}\:{S}_{{n}} \:>\:\mathrm{20} \\ $$$$\Rightarrow\mathrm{S}_{{n}} \:=\:\frac{{a}\left(\mathrm{1}−{r}^{{n}} \right)}{\mathrm{1}−{r}}\:=\:\frac{\mathrm{1}.\left(\mathrm{1}−\left(\mathrm{1}.\mathrm{1}\right)^{{n}} \right.}{\mathrm{1}−\mathrm{1}.\mathrm{1}}\:>\:\mathrm{20} \\ $$$$\frac{\mathrm{1}−\left(\mathrm{1}.\mathrm{1}\right)^{{n}} }{−\mathrm{0}.\mathrm{1}}\:>\:\mathrm{20}\:;\:\left(\mathrm{1}.\mathrm{1}\right)^{{n}} −\mathrm{1}\:>\:\mathrm{2} \\ $$$$\left(\mathrm{1}.\mathrm{1}\right)^{{n}} \:>\:\mathrm{3}\:;\:\mathrm{n}\:>\:\frac{\mathrm{ln}\:\mathrm{3}}{\mathrm{ln}\:\mathrm{1}.\mathrm{1}}\:;\:{n}\:>\:\mathrm{11}.\mathrm{526} \\ $$$$\mathrm{and}\:\mathrm{therefore}\:\mathrm{the}\:\mathrm{smallest}\:\mathrm{number}\: \\ $$$$\mathrm{value}\:\mathrm{of}\:{n}\:\mathrm{is}\:\mathrm{12}\:\blacksquare\: \\ $$

Commented by bemath last updated on 22/Jun/20

thank you both

$$\mathrm{thank}\:\mathrm{you}\:\mathrm{both} \\ $$

Answered by Rio Michael last updated on 22/Jun/20

S_n  = ((a(r^n −1))/(r−1))  S_n  = (((1.1)^n −1)/(1.1−1)) = (((1.1)^n −1)/(0.1))  for S_n  > 20, ⇒ (((1.1)^n −1)/(0.1)) > 20 ⇒ (1.1)^n  > 3  n log (1.1) > log 3  ⇒ n > 11.52 , but n ∈ Z ⇒  n = 12.

$${S}_{{n}} \:=\:\frac{{a}\left({r}^{{n}} −\mathrm{1}\right)}{{r}−\mathrm{1}} \\ $$$${S}_{{n}} \:=\:\frac{\left(\mathrm{1}.\mathrm{1}\right)^{{n}} −\mathrm{1}}{\mathrm{1}.\mathrm{1}−\mathrm{1}}\:=\:\frac{\left(\mathrm{1}.\mathrm{1}\right)^{{n}} −\mathrm{1}}{\mathrm{0}.\mathrm{1}} \\ $$$$\mathrm{for}\:{S}_{{n}} \:>\:\mathrm{20},\:\Rightarrow\:\frac{\left(\mathrm{1}.\mathrm{1}\right)^{{n}} −\mathrm{1}}{\mathrm{0}.\mathrm{1}}\:>\:\mathrm{20}\:\Rightarrow\:\left(\mathrm{1}.\mathrm{1}\right)^{{n}} \:>\:\mathrm{3} \\ $$$${n}\:\mathrm{log}\:\left(\mathrm{1}.\mathrm{1}\right)\:>\:\mathrm{log}\:\mathrm{3} \\ $$$$\Rightarrow\:{n}\:>\:\mathrm{11}.\mathrm{52}\:,\:\mathrm{but}\:{n}\:\in\:\mathbb{Z}\:\Rightarrow\:\:{n}\:=\:\mathrm{12}. \\ $$

Terms of Service

Privacy Policy

Contact: info@tinkutara.com